download scanned papers here - Paper 1 Paper II


UPSC PRELIMS 2019 PAPER I -- ANSWER KEYS




BIRD'S EYEVIEW


  • 2019, a challenge at least as big as 2018, if not bigger : Prelims paper 2019 proved to be a good challenge to anyone – fresh or experienced – who either expected a set pattern based on the year 2018 or even the tough pattern of 2017, rather than being prepared for anything and everything – remember that “uncertainty is the key theme in major competitive exams”. The 2017 papers (both I and II) were quite tough. This affected the cutoffs. And for 2018, the paper was factually tough, though not necessarily as analytically contorted as was the case for 2017. The trend continued in 2019, and most would not have found the paper easy. Lesson (once more) : Steady preparation across topis, going in-depth, no random guesswork, discretion, practice.

  • Summary of major changes :
    1. Topicwise balance same in some areas – (a) International CA and Indian CA (6 and 7 questions in 2019, compared to 9 and 7 in 2018), (b) Environment and Ecology (13 questions compared to 11 for 2018), (c) Geography (5 compared to 6 in 2018)
    2. Topicwise balance changed in some areas – (a) Science and Technology including Agriculture (20 questions compared to 14 asked in 2018), (b) Government schemes reduced to just 1(instead of 6 last year), (c) Modern history reduced to one-third (4 in place of 13 asked in 2018), (d) Polity increased slightly (15 in place of 12 asked in 2018), and (e) Art and Culture went down drastically (1 question compared to 6 last year)
    3. Toughness level remained more or less the same as in 2018, and the ‘strangeness’ part also was retained! Not too many extremely esoteric facts were asked – some were straight
    4. Lot of micro level stuff asked (just like in 2018)
    5. Closely placed options in some questions, making guesstimates a risky game
    6. Trying to “hit more” in order to score more may backfire
    7. Only long term prep helped – no mugging up of “lists”
    8. Total number of sitters that all students must crack : 13 questions. In Set D, these were questions numbered 1 (AIIB), 7 (EODB), 14 (Assets of banks), 17 (Commercial Paper), 22 (PPP), 27 (RBI Data Localisation), 28 (EU GDPR), 42 (Denisovan), 53 (Global Competitiveness Report), 66 (Pyrolysis), 70 (Microbeads), 79 (Atal Innovation Mission) and 84 (dew drops). No student should have missed these ones.
  • Few direct questions : Since 2016 and 2015 were current affairs heavy, and many questions could be solved ONLY by elimination of options, and since many questions were very direct requiring little more than having remembered (or even read) the concept earlier, 2017 was a rude jolt. The 2018 paper continued the streak. And 2019 was no different. If you were prepared for this kind of stuff, the overall composure would have helped. Lesson : It is only a relative performance test, not absolute.

  • Topicwise balance changed : The 2019 paper was dominated by questions from only one area unlike in 2017 and 2018 when Polity had 22 questions and Government Schemes 12 questions. Heaviest this year were : Science and Technology including Agriculture (20 questions), 15 from Indian Economy, 15 from Polity, and 13 from Environment & Ecology. Lesson : Don’t expect anything.

  • Our Support : At PT's IAS Academy, we have tried to incorporate everything possible on as many topics as possible. We wish our students great luck in the results!

  • Easiness limited to the sitters : The 13 questions mentioned above were easy.

  • Government schemes limited : Not too many questions, unlike in 2017.

  • Ambiguous answers : Unlike 2018 and 2017, there were no major ambiguities, but some possible issues of interpretation. Year 2017 had questions like those on Butterfly / Vote / IVC / gypsum etc. This is good news.

  • At this stage, we would like to suggest students to avoid getting into any speculation regarding the "Cut-Offs" for Prelims. A broad guideline could be General Category – 95 to 103, Marks, OBC – almost same and SC/ST – 85 to 92 marks. If you are anywhere close to these, get cracking for Mains preparations. Do not sulk, do not lose hope. Strange things can happen. All the best!



TOPICWISE DISTRIBUTION




DETAILED QUESTIONWISE SOLUTIONS (Set D)


1. With reference to Asian Infrastructure Investment Bank (AIIB), consider the following statements :

  1. AIIB has more than 80 member nations.
  2. India is the largest shareholder in AIIB.
  3. AIIB does not have any members from outside Asia.

Which of the statements given above is / are correct?

  1. 1 only
  2. 2 and 3 only
  3. 1 and 3 only
  4. 1, 2 and 3

Sol. Answer is (a). Only Statement 1 is correct. The Asian Infrastructure Investment Bank (AIIB) – a Chinese initiative – is a multilateral development bank to help build infrastructure in the Asia-Pacific region. Headquartered in Beijing, it began operations in January 2016 and now has grown to 97 approved members worldwide, not just Asia. India is not the largest shareholder, but China is. Clearly, statements 2 and 3 are wrong. Option (a) is thus correct. Reference image(s) here.



2. What was the purpose of Inter-Creditor Agreement signed by Indian banks and financial institutions recently?

  1. To lessen the Government of India's perennial burden of fiscal deficit and current account deficit
  2. To support the infrastructure projects of Central and State Governments
  3. To act as independent regulator in case of applications for loans of Rs. 50 crore or more
  4. To aim at faster resolution of stressed assets of Rs. 50 crore or more which are-under consortium lending

Sol. Answer is (d). The first option is irrelevant as no such thing can be attempted by Indian banks and Financial Institutions. They cannot impact fiscal and current deficits like this! Option (b) sounds legitimate, but is factually wrong. Option (c) is totally wrong. Option (d) is the correct answer. The Inter-Creditor Agreement (ICA) was framed by the Indian Banks’ Association and follows the recommendations of the Sunil Mehta Committee on stressed asset resolution. Initially, 24 lenders including State Bank of India, Bank of India, and Corporation Bank signed, and more joined later. Reference image(s) here.


3. The Chairmen of public sector banks are selected by the

  1. Banks Board Bureau
  2. Reserve Bank of India
  3. Union Ministry of Finance
  4. Management of concerned bank

Answer is (a). The Banks Board Bureau has its genesis in the recommendations of The Committee to Review Governance of Boards of Banks in India, May 2014. The Bureau started functioning from April 01, 2016 as an autonomous recommendatory body, that selects the names for potential Chairmen of Indian PSBs. Shri Bhanu Pratap Sharma was appointed the second Chairman in 2018, replacing former CAG Vinod Rai. Option (b) is wrong, as RBI is the regulator. Option (c) sounds potentially right, but the final appointment is made by the appointments committee of Union Cabinet. That option is not given, so we will go with (a). Option (d) is clearly from dreamland!


4. Consider the following statements:

  1. Petroleum and Natural Gas Regulatory Board (PNGRB) is the first regulatory body set up by the Government of India.
  2. One of the tasks of PNGRB is to ensure competitive markets for gas.
  3. Appeals against the decisions of PNGRB go before the Appellate Tribunals for Electricity.

  1. 1 and 2 only
  2. 2 and 3 only
  3. 1 and 3 only
  4. 1, 2 and 3

Sol. Answer is (b). The Petroleum and Natural Gas Regulatory Board (PNGRB) was constituted under The Petroleum and Natural Gas Regulatory Board Act, 2006 (NO. 19 OF 2006) notified via Gazette Notification dated 31st March, 2006. So, Statement 1 is wrong, as it is definitely not the “first regulatory body” set up by government of India (the statement does not mention in which sector, so we assume all sectors). If Statement 1 is wrong, then options(a), (b) and (d) are wrong. So, (b) alone is right. Statement 2 and 3 are actually correct. The Appellate Tribunal established under section 110 of the Electricity Act, 2003 (36 of 2003) is the Appellate Tribunal for order issued by PNGRB. Reference image(s) here.


5. With reference to communication technologies, what is/are the difference/differences between LTE (Long-Term Evolution) and VoLTE (Voice over Long-Term Evolution)?

  1. LTE is commonly marketed as 3G and VoLTE is commonly marketed as advanced 3G.
  2. LTE is data-only technology and VoLTE is voice-only technology.

Select the correct answer using the code given below.

  1. 1 only
  2. 2 only
  3. Both 1 and 2
  4. Neither 1 nor 2

Sol. Answer is (d). Correct answer is option (d). The term LTE stands for Long Term Evolution and is a registered trademark owned by ETSI (European Telecommunications Standards Institute) for the wireless data communications technology and a development of the GSM/UMTS standards. LTE is commonly marketed as "4G LTE and Advance 4G", hence statement 1 is wrong. So options (a) and (c) are wrong. LTE is also commonly known as 3.95G, and was mainly designed to be data-only technology. Voice over Long-Term Evolution (VoLTE) is a standard for high-speed wireless communication for mobile phones and data terminals — including IoT devices and wearables. VoLTE has up to three times more voice and data capacity than 3G UMTS and up to six times more than 2G GSM. It carries both voice and data, even simultaneously. Hence, statement 2 is also wrong. Hence, option (d) is our choice. Reference image(s) here.


6. Which of the following statements is/are correct regarding the Maternity Benefit (Amendment) Act, 2017?

  1. Pregnant women are entitled for three months pre-delivery and three months
  2. Enterprises with crèches must allow the mother minimum six crèche visits daily.
  3. Women with two children get reduced entitlements.

Select the correct answer using the code given below.

  1. 1 and 2 only
  2. 2 only
  3. 3 only
  4. 1, 2 and 3

Answer is (c). Statement 2 is wrong. The mother is entitled to visit the crèche 4 times a day, not 6 times a day. So, options (a), (b) and (d) are wrong! So, only (c) is correct. If you knew that 2 is wrong, the whole question would be solved instantly. The 2017 amendment to the Maternity Benefit Act of 1961 (MBA), (effective 1st of April, 2017) says that – (i) increased paid maternity leave from 12 weeks to 26 weeks for women employees, unless they have two or more surviving children; (ii) recognition of the rights of an adopting mother and of a commissioning mother (using a surrogate to bear a child) for the first time, who may claim paid maternity leave for 12 weeks; (iii) a “work from home” option that may be of benefit after the maternity leave expires; (iv) and, effective as of the 1st of July, 2017, mandatory crèche (day care) facilities for every establishment employing 50 or more employees, including the right of mothers to visit the crèche four times per day. It is clear from (i) above that statement 3 is correct.


7. Which one of the following is not a sub-index of the World Bank's 'Ease of Doing Business Index'?

  1. Maintenance of law and order
  2. Paying taxes
  3. Registering property
  4. Dealing with construction permits

Answer is (a). A great question, and an expected one too, because Ease of Doing Business is a very hot topic! The World Bank moderated Index includes the following – Starting a business, Dealing with construction permits, Getting electricity, Registering property, Getting credit, Protecting minority investors, Paying taxes, Trading across borders, Enforcing contracts, and Resolving insolvency. Hence, option (a) Maintenance of law and order is not covered. You could have tried to guess it by seeing that (a) is more to do with an internal social-political issue than a business issue. Reference image(s) here.


8. In India, 'extended producer responsibility' was introduced as an important feature in which of the following?

  1. The Bio-medical Waste (Management and Handling) Rules, 1998
  2. The Recycled plastic (Manufacturing and Usage) Rules, 1999
  3. The e-Waste (Management and Handling) Rules, 2011
  4. The Food Safety and Standard Regulations, 2011

Answer is (c). This question was tricky, as you may have found option (b) naturally correct (due to “plastics”). But of the given options, the EPR concept first came in the E-waste (Management and Handling) Rules, 2011, which said that “ … (l) 'extended producer responsibility' means responsibility of any producer of electrical or electronic equipment, for their products beyond manufacturing until environmentally sound management of their end-of-life products; …”. The EPR concept for plastics first came in the Plastic Waste (Management and Handling) Rules, 2011, and not in 1999.


9. The economic cost of food grains to the Food Corporation of India is Minimum Support Price and bonus (if any) paid to the farmers plus

  1. transportation cost only
  2. interest cost only
  3. procurement incidentals and distribution cost
  4. procurement incidentals and charges for godowns

Answer is (c).A tricky question. The FCI (Food Corporation of India) defines Economic Cost as the sum of Acquisition Cost and the Distribution Cost. The closest option is (c). Reference image(s) here.


10. In the context of any country, which one of the following would be considered as part of its social capital?

  1. The proportion of literates in the population
  2. The stock of its buildings, other infrastructure and machines
  3. The size of population in the working age group
  4. The level of mutual trust and harmony in the society

Answer is (d). Options (b) and (d) can be ruled out directly. Now between option (a) and (d), the option (a) is too narrowly defined. Hence, go for (d). Social capital is a concept in social science that involves the potential of individuals to secure benefits and invent solutions to problems through membership in social networks.


download scanned papers here - Paper 1 Paper II


11. The Service Area Approach was implemented under the purview of

  1. Integrated Rural Development Programme
  2. Lead Bank Scheme
  3. Mahatma Gandhi National Rural Employment Guarantee Scheme
  4. National Skill Development Mission

Answer is (b). The Service Area Approach (SAA) is an improved version of Area Approach (AA) of Lead Bank Scheme (LBS). Under SAA plan each commercial bank/Regional Rural Bank (RRB) in rural and semi-urban area is designated to serve 15 to 25 villages for planned and orderly development of rural and semi-urban areas. This scheme was introduced in 1989 and reviewed in December 2004. The Lead Bank Scheme itself was introduced in 1969, envisaging assignment of lead roles to individual banks (public and private) for the districts allotted to them. A bank having a relatively large network of branches in the rural areas of a given district and endowed with adequate financial and manpower resources was made the lead.


12. With reference to the management of minor minerals in India, consider the following statements :

  1. Sand is a 'minor mineral' according to the prevailing law in the country.
  2. State Governments have the power to grant mining leases of minor minerals, but the powers regarding the formation of rules related to the grant of minor minerals lie with the Central Government.
  3. State Governments have the power to frame rules to prevent illegal mining of minor minerals.

Which of the statements given above is / are correct?

  1. 1 and 3 only
  2. 2 and 3 only
  3. 3 only
  4. 1, 2 and 3

Answer is (a). Look at the options – statement 3 is in all four, hence it is correct. If you knew that sand is a minor mineral, then options (b) and (c) too are ruled out. We have to check between options (a) and (d) now. India produces as many as 88 minerals which include 55 minor minerals. According to section 3(e) of the Mines and Minerals (Development and Regulation) Act, 1957 “Minor Minerals” means building stones, gravel, ordinary clay, ordinary sand other than sand used for prescribed purposes, and any other mineral which the Central Government may, by notification in the Official Gazette, declare to be a minor mineral. The word "minerals” includes all minerals except mineral oils- natural gas and petroleum. The central government has the power to notify "minor minerals" under section 3 (e) of the MMDR Act, 1957. On the other hand, as per Section 15 of the MMDR Act, 1957 State Governments have complete powers for making Rules for grant of concessions in respect of extraction of minor minerals and levy and collection of royalty on minor minerals. Hence, statement 2 is wrong. Hence, option (a) is right. Reference image(s) here.


13. Consider the following statements :

  1. Most of India's external debt is owed by governmental entities.
  2. All of India's external debt is denominated in US dollars.

Which of the statements given above is / are correct?

  1. 1 only
  2. 2 only
  3. Both 1 and 2
  4. Neither 1 nor 2

Answer is (d). This is a very good question! Statement 1 is incorrect, as private entities hold most of the external debt. External debt is the money that borrowers in a country owe to foreign lenders. India’s external debt was $510.4 billion at the end of September 2018. Most of it ($ 407.7 b) was owed by private businesses which borrowed at attractive rates from foreign lenders. Statement 2 is also incorrect, as External Debt of India is denominated in Euro, SDR, Japanese Yen, and Pound Sterling apart from US Dollar. The US dollar denominated debt was the largest component of India's external debt with a share of 49.5 per cent at end-March 2018, followed by the Indian rupee (35.8 per cent), SDR (5.5 per cent), Japanese yen (4.8 per cent) and euro (3.4 per cent). Reference image(s) here.


14. Which of the following is not included in the assets of a commercial bank in India?

  1. Advances
  2. Deposits
  3. Investments
  4. Money at call and short notice

Answer is (b). The basic fundamental is that deposits have to be paid back, and hence will be a liability, and not an asset.Major assets of Commercial Banks of India are - Cash in hand with RBI, Money at call and short notice, Investments, Loans, Advances, and Bills discounted and purchased. While major liabilities are Paid-up capital and Reserves, Deposits, Borrowings etc. A simple question indeed!


15. In the context of India, which of the following factors is/are contributor/contributors to reducing the risk of a currency crisis?

  1. The foreign currency earnings of India's IT sector
  2. Increasing the government expenditure
  3. Remittances from Indians abroad

Select the correct answer using the code given below.

  1. 1 only
  2. 1 and 3 only
  3. 2 only
  4. 1, 2 and 3

Answer is (b). A ‘currency crisis’ is a situation when the Indian rupee abruptly starts losing value, and depreciates rapidly. That puts pressure on the external account. This happens when more dollars flow out of the system (generally). So anything that can bring in dollars will help reduce the risk of a currency crisis. Indian IT sector earnings have the direct contribution in bringing in dollars via the Invisible Account, so statement 1 is correct. Remittances from abroad are most welcome, and India is no. 1 recipient of it, so statement 3 is correct. Statement 2 has no link directly with the question. Answer is (b). Reference image(s) here.


16. Which one of the following suggested that the Governor should be an eminent person from outside the State and should be a detached figure without intense political links or should not have taken part in politics in the recent past?

  1. First Administrative Reforms Commission (1966)
  2. Rajamannar Committee (1969)
  3. Sarkaria Commission (1983)
  4. National Commission to Review the Working of the Constitution (2000)

Answer is (c). Sarkaria Commission recommended it. It said that – 1. On the qualification of a governor - He should be eminent in some walk of life, He should be a person from outside the state, He should be a detached figure without intense political links, or should not have taken part in politics in recent past, He should not be a member of the ruling party, etc.




17. Which of the following is issued by registered foreign portfolio investors to overseas investors who want to be part of the Indian stock market without registering themselves directly?

  1. Certificate of Deposit
  2. Commercial Paper
  3. Promissory Note
  4. Participatory Note

Answer is (d). This topic has been in news recently. Let us see all – 1. Certificate of Deposit (CD) is a negotiable money market instrument and issued in dematerialised form or as a Promissory Note against funds deposited at a bank or other eligible financial institution for a specified time period. It was introduced in India in 1989. 2. Commercial Paper (CP) is an unsecured money market instrument issued in the form of a promissory note. 3. A promissory note means a signed document containing a written promise to pay a stated sum to a specified person at a specified date or on demand. 4. Participatory notes (P-Notes or PNs) are financial instruments required by investors or hedge funds to invest in Indian securities without having to register with the Securities and Exchange Board of India (SEBI). P-Notes are among the group of investments considered to be Offshore Derivative Investments (ODIs). Reference image(s) here.


18. Consider the following statements :

  1. As per law, the Compensatory Afforestation Fund Management and Planning Authority exists at both National and State levels.
  2. People's participation is mandatory in the compensatory afforestation programmes carried out under the Compensatory Afforestation Fund Act, 2016.

Which of the statements given above is / are correct?

  1. 1 only
  2. 2 only
  3. Both 1 and 2
  4. Neither 1 nor 2

Answer is (a). Statement 1 is correct. Compensatory Afforestation Fund Act, 2016, Chapter III, section 8 states about the national authority and section 9 states about state authority.


Statement 2 is incorrect. As there is no mention of the mandatory participation of people in the programs carried out under it, in the act. The key word was “mandatory”.

The Supreme Court in January 2019 directed that all the funds worth around Rs 54,000 crore under the Compensatory Afforestation Fund Management and Planning Authority (Campa) be transferred to a new Authority created by the central government, setting the stage for its utilisation by both the central and state governments for its green initiatives. Reference image(s) here.


19. In India, which of the following review the independent regulators in sectors like telecommunications, insurance, electricity, etc.?

  1. Ad Hoc Committees set up by the Parliament
  2. Parliamentary Department Related Standing Committees
  3. Finance Commission
  4. Financial Sector Legislative Reforms Commission
  5. NITI Aayog

Select the correct answer using the code given below.

  1. 1 and 2
  2. 1, 3 and 4
  3. 3, 4 and 5
  4. 2 and 5

Answer is (a). Look at the statements carefully. Can the NITI Aayog (a non-statutory, non-constitutional body) ever be given such a massive responsibility? No. Hence 5 cannot be part of the answer. Similarly, the Finance Commission has nothing to do with regulators! Hence, 3 also is ruled out. So all options are ruled out except (a).
In India, the Parliament scrutinises regulators by the following means: (i) question hour; (ii) discussions in Parliament; and (iii) parliamentary committees (Department Related Standing Committees, Finance Committees like (i) the Committee on Estimates; and (ii) the Public Accounts Committee (PAC), Ad Hoc Committees). Other means of legislative oversight, such as annual submission of reports by regulators to Parliament, are sometimes provided in enactments or are laid down in the Rules of Procedures of the Parliament.


20. With reference to India's Five-Year Plans, which of the following statements is/are correct?

  1. From the Second Five-Year Plan, there was a determined thrust towards substitution of basic and capital good industries.
  2. The Fourth Five-Year Plan adopted the objective of correcting the earlier trend of increased concentration of wealth and economic power.
  3. In the Fifth Five-Year Plan, for the first time, the financial sector was included as an integral part of the Plan.

Select the correct answer using the code given below.

  1. 1 and 2 only
  2. 2 only
  3. 3 only
  4. 1, 2 and 3

Answer is (a). All three are correct. In statement 1, we assume it to mean that India was building its own infrastructure of basic and capital good industries. For statement 2, the Planning Commission said about the 4th Five Year Plan that it was a conscious, internally consistent and carefully thought-out programme to raise the standard of living of the people, especially of the less privileged sections of society. The overriding inspiration must be a burning sense of social justice. While increased production is of the utmost importance it is equally important to remove or reduce, and prevent the concentration of wealth and economic power.
(http://planningcommission.nic.in/plans/planrel/fiveyr/4th/4ppre.htm) Regarding statement 3, no explicit mention is found.


21. With reference to the Constitution of India, consider the following statements:

  1. No High Court shall have the jurisdiction to declare any central law to be constitutionally invalid.
  2. An amendment to the Constitution of India cannot be called into question by the Supreme Court of India.

Which of the statements given above is / are correct?

  1. 1 only
  2. 2 only
  3. Both 1 and 2
  4. Neither 1 nor 2

Answer is (d). Statement 2 is obviously wrong, and the Supreme Court can question any and every amendment to the Constitution if it finds the same violating the Basic Structure Doctrine. The best example is that of the 99th amendment, 2014 (NJAC judgment). Hence, options (b) and (c) are wrong. Now, statement 1 is also wrong as there is no such restriction on any High Court of India. The present position of law, as expressly upheld by various high court judgments relying on Kusum Ingots & Alloys Ltd. v. Union of India, and as impliedly accepted by the Supreme Court, is that interim or final orders passed by a High Court on the constitutional validity of a Central Legislation is applicable throughout the country and hence binds other High Courts from taking a contrary view on the issue. Though this principle may not have the constitutional sanction of Article 226, the same is the law of the land for all practical purposes.


22. Consider the following statements

  1. Purchasing Power Parity (PPP) exchange rates are calculated by comparing the prices of the same basket of goods and services in different countries.
  2. In terms of PPP dollars, India is the sixth largest economy in the world.

Which of the statements given above is / are correct?

  1. 1 only
  2. 2 only
  3. Both 1 and 2
  4. Neither 1 nor 2

Answer is (a). Statement 1 is correct, by definition. But statement 2 is wrong, as Indian economy is now ranked third in the world on PPP dollar basis. Hence, answer is (a). Reference image(s) here.


23. With reference to the cultivation of Kharif crops in India in the last five years, consider the following statements :

  1. Area under rice cultivation is the highest.
  2. Area under the cultivation of jowar is more than that of oilseeds.
  3. Area of cotton cultivation is more than that of sugarcane.
  4. Area under sugarcane cultivation has steadily decreased.

Which of the statements given above are correct?

  1. 1 and 3 only
  2. 2, 3 and 4 only
  3. 2 and 4 only
  4. 1, 2, 3 and 4

Answer is (a). Statement 2 is wrong, as sowing area for jowar (one of the coarse cereals) is definitely not more than that for oilseeds. Coarse cereals are a broad sub-group of several short duration warm weather (Kharif) crops such as Jowar (Sorghum), Bajra (Pearl Millet), Maize, Ragi (Finger Millet) etc. So, if 2 is wrong, then automatically the only answer is (a). Both 1 and 3 are correct. Reference image(s) here.
(Full latest data here – http://agricoop.gov.in/sites/default/files/2ndADVEST201819_E.pdf)


24. Among the agricultural commodities imported by India, which one of the following accounts for the highest imports in terms of value in the last five years?

  1. Spices
  2. Fresh fruits
  3. Pulses
  4. Vegetable oils

Answer is (d). Indian edible oil import has grown at 8.4% CAGR over the last decade. Broadly, the portion of refined oil in the imports has been ~15% of the total imports. The remaining crude (non-refined) edible oil imported is refined in local units. If Indian yield improves from existing case to even global average yields, Indian edible oil production would grow by 67% to ~13.5 mn tons on the same area of cultivation. If yields improve to global best rates, Indian edible oil production would become 3x the existing 8-9 mn tons production and India will be self-sufficient on existing cultivated land itself! Regarding pulses : Two straight years of drought pushed up pulse prices in 2015 and forced New Delhi to allow duty-free imports. But record imports of 6.6 million tonnes in the 2016-17 fiscal year led to a crash in local prices. As part of its efforts to curb imports, the government started fixing import quotas in 2018. India is the world's biggest producer and consumer of pulses. Reference image(s) here.


25. In the context of polity, which one of the following would you accept as the most appropriate definition of liberty?

  1. Protection against the tyranny of political rulers
  2. Absence of restraint
  3. Opportunity to do whatever one likes
  4. Opportunity to develop oneself fully.

Answer is (b). In political science, the word “Liberty” stands derived from the Latin word 'Liber” which means 'free'. Broadly speaking, liberty is the ability to do as one pleases. But it cannot be taken to mean "to do whatever one likes" as that could lead to anarchy. So option (c) is wrong. Liberty definitely is freedom from restraints that bind individuals against their wishes. So option (b) is most correct. Option (d) is also not wrong, but (c) is better. Option (a) is ruled out as it is an implied utility of liberty and not a definition of it.


26. Which one of the following is not the most likely measure the Government/RBI takes to stop the slide of Indian rupee?

  1. Curbing imports of non-essential goods-and promoting exports
  2. Encouraging Indian borrowers to issue rupee denominated Masala Bonds
  3. Easing conditions relating to external commercial borrowing
  4. Following an expansionary monetary policy

Answer is (d). If the Indian rupee is sliding, then option (a) is one of the first steps to be taken. That stops unnecessary spend of precious foreign exchange. Option (b) is a good idea as well as dollar borrowings are avoided. Option (c) is also a preferred route, and gets more dollars in (it was done in 2018). Option (d) is not directly linked to the problem at hand. An expansionary monetary policy means reduction in interest rates (softening). That means more possible credit offtake and credit flow in the local system, which may lead to a depreciation of rupee’s value further (or it may not).


27. Consider the following statements:

The Reserve Bank of India's recent directives relating to 'Storage of Payment System Data', popularly known as data diktat, command the payment system providers that

  1. they shall ensure that entire data relating to payment systems operated by them are stored in a system only in India
  2. they shall ensure that the systems are owned and operated by public sector enterprises
  3. they shall submit the consolidated system audit report to the Comptroller and Auditor General of India by the end of the calendar year

Which of the statements given above is/are correct?

  1. 1 only
  2. 1 and 2 only
  3. 3 only
  4. 1, 2 and 3

Answer is (a). Statement 2 is clearly wrong, as no such order was issued. So, options (b) and (d) are wrong. Statement 3 is also wrong. Hence answer is (a) – 1 only. The payment system providers shall ensure that entire data relating to payment systems operated by them are stored in a system only in India. This is what make Donald Trump administration very upset about India.


28. Which of the following adopted a law on data protection and privacy for its citizens known as 'General Data Protection Regulation' in April 2016 and started implementation of it from 25th May, 2018?

  1. Australia
  2. Canada
  3. The European Union
  4. The United States of America

Answer is (c). Perhaps one of the easiest questions! The European Union brought the GDPR in 2018 and totally changed the rules governing digital enterprises and data management. Reference image(s) here.


29. Recently, India signed a deal known as 'Action Plan for Prioritization and Implementation of Cooperation Areas in the Nuclear Field' with which of the following countries?

  1. Japan
  2. Russia
  3. The United Kingdom
  4. The United States of America

Answer is (b). During the visit of H. E. Vladimir Putin, President of the Russian Federation to India, Action Plan for Prioritization and Implementation of Co-operation Areas in the Nuclear Field Identified Jointly by India and Russia was signed on 5th October, 2018 in New Delhi. Civil nuclear cooperation between India and Russia is an important component of strategic partnership contributing to India’s energy-security and its commitments under the Paris Agreement on Climate Change.


30. The money multiplier in an economy increases with which one of the following?

  1. Increase in the cash reserve ratio
  2. Increase in the banking habit of the population
  3. Increase in the statutory liquidity ratio
  4. Increase in the population of the country

Answer is (b). If you go by the definition, then money multiplier is the ratio of commercial bank money to central bank money under a fractional-reserve banking system. The actual ratio of money to central bank money, also called the money multiplier, is lower because some funds are held by the non-bank public as currency etc. Hence, if the banking habit of population changes, and they enter mainstream banking (more accounts, etc.), then the multiplier will increase. Hence, (b). Options (a) and (c) will have the reverse effect. Option (d) does not indicate anything meaningful. Reference image(s) here.


download scanned papers here - Paper 1 Paper II


31. Consider the following statements about Particularly Vulnerable Tribal Groups (PVTGs) in India :

  1. PVTGs reside in 18 States and one Union Territory.
  2. A stagnant or declining population is one of the criteria for determining PVTG status.
  3. There are 95 PVTGs officially notified in the country so far.
  4. Irular and Konda Reddi tribes are included in the list of PVTGs.

Which of the statements given above are correct?

  1. 1, 2 and 3
  2. 2, 3 and 4
  3. 1, 2 and 4
  4. 1, 3 and 4

Answer is (c). Statement 1 is correct as PVTGs reside in 18 States and UT of Andaman and Nicobar Islands. It means answer cannot be option (b). Statement 3 is incorrect as there are 75 PVTGs officially notified in the country so far. So (a) and (d) are ruled out. Hence, our final answer is (c). Reference image(s) here.



32. With reference to the Constitution of India, prohibitions or limitations or provisions contained in ordinary laws cannot act as prohibitions or limitations on the constitutional powers under Article 142. It could mean which one of the following?

  1. The decisions taken by the Election Commission of India while discharging its duties cannot be challenged in any court of law.
  2. The Supreme Court of India is not constrained in the exercise of its powers by laws made by the Parliament.
  3. In the event of grave financial crisis in the country, the President of India can declare Financial Emergency without the counsel from the Cabinet.
  4. State Legislatures cannot make laws on certain matters without the concurrence of Union Legislature.

Answer is (b). Article 142 of Constitution of India gives wide powers to the Supreme Court for enforcement of its decrees and orders. Option (a) is irrelevant. Option (c) is irrelevant as that is covered under Article 360. Option (d) is also irrelevant, and is covered under Article 246. If you only remembered the articles and what they relate to, you could have eliminated three options directly. Reference image(s) here.


33. With reference to the Legislative Assembly of a State in India, consider the following statements:

  1. The Governor makes a customary address to Members of the House at the commencement of the first session of the year.
  2. When a State Legislature does not have a rule on a particular matter, it follows the Lok Sabha rule on that matter.

Which of the statements given above is / are correct?

  1. 1 only
  2. 2 only
  3. Both 1 and 2
  4. Neither 1 nor 2

Answer is (c). Article 176(1) of the Constitution of India enjoins that the Governor shall Address both the Houses assembled together at the commencement of the first Session after each general election to the Assembly and at the commencement of the first session of each year and inform the Legislature of the causes of its Summons. Hence (1) is true. So options (b) and (d) are wrong. Now check statement 2. That is also correct, by convention. Hence, answer is (c).


34. Consider the following statements :

  1. The United Nations Convention against Corruption (UNCAC) has a 'Protocol against the Smuggling of Migrants by Land, Sea and Air'.
  2. The UNCAC is the ever-first legally binding global anti-corruption instrument.
  3. A highlight of the United Nations Convention against Transnational Organized Crime (UNTOC) is the inclusion of a specific chapter aimed at returning assets to their rightful owners from whom they had been taken illicitly.
  4. The United Nations Office on Drugs and Crime (UNODC) is mandated by its member States to assist in the implementation of both UNCAC and UNTOC.

Which of the statements given above are correct?

  1. 1 and 3 only
  2. 2, 3 and 4 only
  3. 2 and 4 only
  4. 1, 2, 3 and 4

Answer is (c). Statement 1 is incorrect as UN Convention against Transnational Organized Crime (UNTOC) has the Protocol against the Smuggling of Migrants by Land, Sea and Air. So, options (a) and (d) are wrong. Now check statement 3. It is true for United Nations Convention against Corruption (UNCAC) and not for UNTOC as given. So it is incorrect. So, option (b) is wrong. Hence, (c) is correct. This question was tricky, and was asked in context of global terrorism, FATF related developments, global corruption cases etc. Reference image(s) here.


35. Consider the following statements:

  1. As per recent amendment to the Indian Forest Act, 1927, forest dwellers have the right to fell the bamboos grown on forest areas.
  2. As per the Scheduled Tribes and Other Traditional Forest Dwellers (Recognition of Forest Rights) Act, 2006, bamboo is a minor forest produce.
  3. The Scheduled Tribes and Other Traditional Forest Dwellers (Recognition of Forest Rights) Act, 2006 allows ownership of minor forest produce to forest dwellers.

Which of the statements given above is / are correct?

  1. 1 and 2 only
  2. 2 and 3 only
  3. 3 only
  4. 1, 2 and 3

Answer is (d). All three statements are correct. After amending Section 2(7) of Indian Forest Act, 1927, bamboo is no longer a tree and felled bamboo is not timber (earlier bamboo was considered timber). So any bamboo grown in private or homestead land by millions of farmers does not require a felling permission or transit permission from any state forest dept. So statement 1 is correct. Statement 2 – Under the Scheduled Tribes and Other Traditional Forest Dwellers (Recognition of Forest Rights) Act, 2006 “minor forest produce” includes all non-timber forest produce of plant origin including bamboo, brush wood, stumps, cane, tussar, cocoons, honey, wax, lac, tendu or kendu leaves, medicinal plants and herbs, roots, tubers and the like. So statement 2 is also correct. Statement 3 is also correct. Reference image(s) here.


36. Which Article of the Constitution of India safeguards one's right to marry the person of one's choice?

  1. Article 19
  2. Article 21
  3. Article 25
  4. Article 29

Answer is (b). In context of honour killings, Shakti Vahini, an NGO, had approached the Indian Supreme Court, seeking directions to the State Governments and the Central Government to take preventive steps to combat honour crimes. A bench of the Supreme Court of India said in 2018 in a verdict that: “Honour killing guillotines individual liberty, freedom of choice and one’s own perception of choice. It has to be sublimely borne in mind that when two adults consensually choose each other as life partners, it is a manifestation of their choice which is recognized under Articles 19 and 21 of the Constitution. Such a right has the sanction of the constitutional law and once that is recognized, the said right needs to be protected…”.
The key word in this question is “safeguard”. While Article 19 gives a citizen this freedom, it is Article 21 that will safeguard that freedom. The Right to Life and Liberty says that “No person shall be deprived of his life or personal liberty except according to a procedure established by law.”


37. Consider the following statements :

  1. According to the Indian Patents Act, a biological process to create a seed can be patented in India.
  2. In India, there is no Intellectual Property Appellate Board.
  3. Plant varieties are not eligible to be patented in India.

Which of the statements given above is/are correct?

  1. 1 and 3 only
  2. 2 and 3 only
  3. 3 only
  4. 1, 2 and 3

Answer is (c). Statement 2 is wrong, as the IPAB does exist, and has been in news in some high-profile cases. The Intellectual Property Appellate Board (IPAB) of India was constituted on September 15, 2003 by the Indian Government to hear and resolve the appeals against the decisions of the registrar under the Indian Trademarks Act, 1999 and the Geographical Indications of Goods (Registration and Protection) Act, 1999. Hence, options (b) and (d) are ruled out. Now, Statement 1 is not correct. Refer to the point (j) on page 9 of The Patent Act, 1970. Statement 3 is correct. After India signed the TRIPS under WTO, it enacted the Protection of Plant Varieties and Farmers' Rights Act, 2001 (PPV&FR Act 2001) to offer IP protection to plant breeders, researchers and farmers who have developed any new or extant plant varieties. India does not grant patents for plant varieites, but has this sui generis (unique) system which is more convenient to Indian farmers and breeders. Hence, our answer is (c). Reference image(s) here.


38. Consider the following statements :


The Environment Protection Act, 1986 empowers the Government of India to

  1. state the requirement of public participation in the process of environmental protection, and the procedure and manner in which it is sought
  2. lay down the standards for emission or discharge of environmental pollutants from various sources

Which of the statements given above is/ are correct?

  1. 1 only
  2. 2 only
  3. Both 1 and 2
  4. Neither 1 nor 2

Answer is (b). Statements 1 is not correct, as it is not mentioned anywhere in the EPA 1986. Statement 2 is indeed correct. Refer to Chapter II of the Environment Protection Act, 1986. Reference image(s) here.


39. As per the Solid Waste Management Rules, 2016 in India, which one of the following statements is correct?

  1. Waste generator has to segregate waste into five categories.
  2. The Rules are applicable to notified urban local bodies, notified towns and all industrial townships only.
  3. The Rules provide for exact and elaborate criteria for the identification of sites for landfills and waste processing facilities.
  4. It is mandatory on the part of waste generator that the waste generated in one district cannot be moved to another district.

Answer is (c). All waste generators shall segregate and store the waste generated by them in three separate streams namely bio‐degradable, non bio‐degradable and domestic hazardous wastes in suitable bins and handover segregated wastes to authorized rag‐pickers or waste collectors. Option (a) is not correct. The Rules shall apply to every urban local body, outgrowths in urban agglomerations, census towns as declared by the Registrar General and Census Commissioner of India, notified areas, notified industrial townships, areas under the control of Indian Railways, airports, airbase, Port and harbour, defense establishments, special economic zones, State and Central government organizations, places of pilgrims, religious & historical importance as may be notified by respective state government from time to time and to every domestic, institutional, commercial and any other non residential solid waste generator except industrial waste, hazardous waste, hazardous chemicals, bio medical wastes, e‐waste, lead acid batteries and radio‐active waste. Option (b) is not correct. In the law there is not mention of inter-district movement. Refer the file. Option (c) is correct. Reference image(s) here.


40. Consider the following statements :

As per the Industrial Employment (Standing Orders) Central (Amendment) Rules, 2018

  1. if rules for fixed-term employment are implemented, it becomes easier for the firms/companies to lay off workers.
  2. no notice of termination of employment shall be necessary in the case of temporary workman.

Which of the statements given above is/are correct?

  1. 1 only
  2. 2 only
  3. Both 1 and 2
  4. Neither 1 nor 2

Answer is (c). It is clear that to introduce more flexibility in hiring and firing, the norms were changed. Statement 1 is correct. In paragraph 13 for sub- paragraph (b) of Model Standing Order, the following is introduced "No workman employed on fixed term employment basis as a result of non-renewal of contract or employment or on the expiry of such contract period without it being renewed,shall be entitled to any notice or pay in lieu thereof, if his services are terminated." So, statement 2 is also correct. Answer is (c).

download scanned papers here - Paper 1 Paper II


41. In the context of digital technologies for entertainment, consider the following statements:

  1. In Augmented Reality (AR), a simulated environment is created and the physical world is completely shut out.
  2. In Virtual Reality (VR), images generated from a computer are projected onto real-life objects or surroundings.
  3. AR allows individuals to be present in the world and improves the experience using the camera of smart-phone or PC.
  4. VR closes the world, and transposes an individual, providing complete immersion experience.

Which of the statements given above is / are correct?

  1. 1 and 2 only
  2. 3 and 4
  3. 1, 2 and 3
  4. 4 only

Answer is (b). First, we must recall that Virtual Reality is all about creating an alternate world for the user through total immersion in the VR world. Hence, statement 4 is definitely correct. Hence, options (a) and (c) are wrong. Augmented reality is an enhanced version of reality created by the use of technology to add digital information on an image of something. Now, check Statement 3. It is also correct. Hence, best answer is option (b). Reference image(s) here.


42. The word 'Denisovan' is sometimes mentioned in media in reference to

  1. fossils of a kind of dinosaurs
  2. an early human species
  3. a cave system found in North-East India
  4. a geological period in the history of Indian subcontinent

Answer is (b). This term relates to a now-extinct species or subspecies of archaic humans in the genus Homo. It made news over the past one year as fossils were found in Tibet and dated to more than 1.5 lakh years ago. This question could be solved by regular newspaper reading. PT education’s famous “Current Affairs with PT” series covered it in May 2019. Reference image(s) here.


43. With reference to the recent developments in science, which one of the following statements is not correct?

  1. Functional chromosomes can be created by joining segments of DNA taken from cells of different species.
  2. Pieces of artificial functional DNA can be created in Iaboratories.
  3. A piece of DNA taken out from an animal cell can be made to replicate outside a living cell in a laboratory.
  4. Cells taken out from plants and animals can be made to undergo cell division in laboratory petri dishes.

Answer is ( - ). Recombinant DNA (rDNA) molecules are DNA molecules formed by laboratory methods of genetic recombination (such as molecular cloning) to bring together genetic material from multiple sources, creating sequences that would not otherwise be found in the genome. Recombinant DNA molecules are sometimes called chimeric DNA, because they can be made of material from two different species, like the mythical chimera. So, option (a) is correct. In May 2019, scientists at Cambridge University claimed to have created a living organism with human-made DNA - the bacteria E. Coli. The entire genome was synthetic. So, (b) is also correct. Option (c) is also correct because it is regularly done (one reference - https://courses.lumenlearning.com/boundless-biology/chapter/dna-replication/). Option (d) is also correct. Reference image(s) here.


44. Consider the following statements:

A digital signature is

  1. an electronic record that identifies the certifying authority issuing it
  2. used to serve as a proof of identity of an individual to access information or server on Internet.
  3. an electronic method of signing an electronic document and ensuring that the original content is unchanged

Which of the statements given above is / are correct?

  1. 1 only
  2. 2 and 3 only
  3. 3 only
  4. 1, 2 and 3

Answer is (b). A digital signature is an electronic form of a signature that can be used to authenticate the identity of the sender of a message or the signer of a document, and also ensure that the original content of the message or document that has been sent is unchanged. Digital signatures are easily transportable and cannot be imitated by someone else. The ability to ensure that the original signed message arrived means that the sender cannot easily disclaim it later. https://www.e-mudhra.com/faq.html) So, Statements 2 and 3 are correct. But, digital signatures are different from "Digital signature certificates" (DSCs). A DSC is the electronic format of physical or paper certificate like a driving License, passport etc. A digital signature is an electronic method of signing an electronic document whereas a Digital Signature Certificate is a computer based record that identifies the Certifying Authority issuing it. Hence, statement 1 is wrong. So, correct answer is option (b).


45. In the context of wearable technology, which of the following tasks is/are accomplished by wearable devices?

  1. Location identification of a person
  2. Sleep monitoring of a person
  3. Assisting the hearing impaired person

Select the correct answer using the code given below.

  1. 1 only
  2. 2 and 3 only
  3. 3 only
  4. 1, 2 and 3

Answer is (d). Wearable technology does all the things mentioned. It is fashion technology, tech togs, or fashion electronics in the form of smart electronic devices (electronic device with micro-controllers) that can be incorporated into clothing or worn on the body as implants or accessories. Wearable devices such as activity trackers act as an Internet of Things. It has a wide variety of applications including location identification (via GPS), sleep monitoring, hearing aid etc. The field is expanding rapidly.


46. 'RNA interference (RNAi)' technology has gained popularity in the last few years. Why?

  1. It is used in developing gene silencing therapies.
  2. It can be used in developing therapies for-the treatment of cancer.
  3. It can be used to develop hormone replacement therapies.
  4. It can be used to produce crop plants that are resistant to viral pathogens.

Select the correct answer using the code given below.

  1. 1, 2 and 4
  2. 2 and 3
  3. 1 and 3
  4. 1 and 4 only

Answer is (a). RNA interference (RNAi) is a biological process in which RNA molecules neutralize targeted mRNA molecules, and inhibit gene expression (gene silencing). Technically speaking, RNA interference (RNAi) or Post-Transcriptional Gene Silencing (PTGS) is a conserved biological response to double-stranded RNA that mediates resistance to both endogenous parasitic and exogenous pathogenic nucleic acids, and regulates the expression of protein-coding genes. This natural mechanism for sequence-specific gene silencing promises to revolutionize experimental biology and has important practical applications in functional genomics, therapeutic intervention, agriculture and other areas. Except 3, all other statements are correct.

[ RNA and DNA are nucleic acids, and, along with lipids, proteins and carbohydrates, constitute the four major macromolecules essential for all known forms of life. Like DNA, RNA is assembled as a chain of nucleotides, but unlike DNA it is more often found in nature as a single-strand folded onto itself, rather than a paired double-strand. ]




47. Recently, scientists observed the merger of giant 'blackholes' billions of light-years away from the Earth. What is the significance of this observation?

  1. 'Higgs boson particles' were detected.
  2. 'Gravitational waves' were detected.
  3. Possibility of inter-galactic space travel through 'wormhole' was confirmed.
  4. It enabled the scientists to understand 'singularity'.

Answer is (b). In December 2018, scientists found that billions of light years away, two black holes had collided to create a larger one – the biggest black hole merger yet detected. It had a mass more than 80 times that of the sun. The resulting energy injected into the fabric of spacetime was also record breaking, with five sun’s worth of mass released in the form gravitational waves as the two holes spiralled in towards each other. A description of the event, along with three other smaller separate black hole collisions, was released as part of a collection of detections by the Advanced Laser Interferometer Gravitational-Wave Observatory (aLIGO) in the US and the Advanced Virgo facility in Italy. The facility helps detect gravitational waves. You can rule out option (c) directly, as it is pure science-fiction. Option (a) is not related. Reference image(s) here. Reference image(s) here.


48. Which of the following are the reasons for the occurrence of multi-drug resistance in microbial pathogens in India?

  1. Genetic predisposition of some people
  2. Taking incorrect doses of antibiotics to cure diseases
  3. Using antibiotics in livestock farming
  4. Multiple chronic diseases in some people

Select the correct answer using the code given below.

  1. 1 and 2
  2. 2 and 3 only
  3. 1, 3 and 4
  4. 2, 3 and 4

Answer is (b). The rampant use (misuse, abuse) of antibiotic drugs in India (and across the world) has created a dangerous situation of AMR or ABR. Large amounts of antibiotics used for human therapy, as well as for farm animals and even for fish in aquaculture, resulted in the selection of pathogenic bacteria resistant to multiple drugs. Antimicrobial resistance (AMR or AR) is the ability of a microbe to resist the effects of medication that once could successfully treat the microbe. The term antibiotic resistance (AR or ABR) is a subset of AMR, as it applies only to bacteria becoming resistant to antibiotics. Multidrug-resistant (MDR) bacteria become resistant to more than one antibiotic. This can occur in two distinct ways – (i) a bacterium can have several different resistance genes, each providing resistance to a particular antibiotic, or (ii) a single resistance mechanism gives resistance to more than one antibiotic. Of the given statements, 3 is definitely correct. Hence, option (a) is ruled out. AMR or MDR is not due to individual composition of any patient, but due to system failure (excessive or incorrect use).


49. What is Cas9 protein that is often mentioned in news?

  1. A molecular scissors used in targeted gene editing
  2. A biosensor used in the accurate detection of pathogens in patients
  3. A gene that makes plants pest-resistant
  4. A herbicidal substance synthesized in genetically modified crops

Answer is (a). The Cas9 (CRISPR associated protein 9) is extensively used in genetic engineering applications. Its main function is to cut DNA and therefore it can alter a cell's genome. To survive in a variety of challenging, inhospitable habitats that are filled with bacteriophages, bacteria and archea have evolved methods to evade and fend off predatory viruses. This includes the CRISPR system of adaptive immunity. Cas9 is an RNA-guided DNA endonuclease enzyme associated with the CRISPR (Clustered Regularly Interspaced Short Palindromic Repeats) adaptive immunity system in Streptococcus pyogenes, among other bacteria. Reference image(s) here.


50. Which one of the following statements is not correct?

  1. Hepatitis B virus is transmitted much like HIV.
  2. Hepatitis B, unlike Hepatitis C, does not have a vaccine.
  3. Globally, the number of people infected with Hepatitis B and C viruses are several times more than those infected with HIV.
  4. Some of those infected with Hepatitis Band C viruses do not show the symptoms for many years.

Answer is (b). The vaccine is available for Hepatitis B, and not Hepatitis C. Hence (b) is wrong, and that is our answer. Reference image(s) here.

download scanned papers here - Paper 1 Paper II




51. With reference to Mughal India, what is/are the difference/differences between Jagirdar and Zamindar?

  1. Jagirdars were holders of land assignments in lieu of judicial and police duties, whereas Zamindars were holders of revenue rights without obligation to perform any duty other than revenue collection.
  2. Land assignments to Jagirdars were hereditary and revenue rights of Zamindars were not hereditary.

Select the correct answer using the code given below.

  1. 1 only
  2. 2 only
  3. Both 1 and 2
  4. Neither 1 nor 2

Answer is (a). Jagirdar and Zamindar were the two types of intermediaries that collected the land revenue for the king during the Mughal period.

Jagirdars:They were the king's officials who enjoyed the land gifted from the king. They were the rank (Mansab) holders given by the king called Mansabdar. They were allotted non-inheritable land area equivalent to their fee amount called Jagir. So statement 2 is wrong. Answer can be (a) or (d) only. The Mansab allotted were not the same for each year and revised from time to time i.e., Jagirdar got promotion or demotions. Jagirdar having more land area under him was more powerful and enjoyed the police jurisdiction as well. Office of jagirdar came into existence after the advent of Mansabdari system (Indian varient of feudal system) during Akbar's reign (though similar type of officials existed in delhi-sultanete period too). Jagirdars kept a legion of soldiers and were responsible for their salaries and other needs.They were usually transferred regularly every 3-4 years lest someone assert sovereignty over alloted Jagir.

Zamindar:The holder of the land was the Zamindar. They had the hereditary right over the land to claim share in peasant's output other than land revenues. In most of the mughal period the Zamindar collected land revenue from the actual tillers of land and for this they were allowed one-tenth of the collections either in cash or an equivalent amount of tax free land. Thus, during the Mughal period zamindars and peasants held villages were found side-by-side. The revenue is fixed by the kingdom but the amount payable, by the peasants depends on the discretion's of the jagirdars and zamindars.


52. With reference to land reforms in independent India, which one of the following statements is correct?

  1. The ceiling laws were aimed at family holdings and not individual holdings.
  2. The major aim of land reforms was providing agricultural land to all the landless.
  3. It resulted in cultivation of cash crops as a predominant form of cultivation.
  4. Land reforms permitted no exemptions to the ceiling limits.

Answer is (b). The important step taken under land reforms in India was the imposition of the land ceiling. In the first phase till 1972, the reforms focussed on the total amount of land an individual could hold. Later, the unit was made “a family” and not “an individual”. Option (a) is wrong. The law implemented the fixation of the ceiling on landholding, and allowed the government to take over the surplus land. Such land was then distributed among landless farmers or small farmers. The imposition of such a ceiling was to deter the concentration of land in the hands of a few. Option (b) is correct. Options (c) is wrong. Option (d) is wrong as exemptions were given to different sectors, across states, like education, IT, plantations etc.


53. The Global Competitiveness Report is published by the

  1. International Monetary Fund
  2. United Nations Conference on Trade and Development
  3. World Economic Forum
  4. World Bank

Answer is (c). The World Economic Forum (WEF) publishes the report annually. A direct and simple question. It was first published in 2004. It ranks countries based on the Global Competitiveness Index, developed by Xavier Sala-i-Martin and Elsa V. Artadi. The report "assesses the ability of countries to provide high levels of prosperity to their citizens". This in turn depends on how productively a country uses available resources. Therefore, the Global Competitiveness Index measures the set of institutions, policies, and factors that set the sustainable current and medium-term levels of economic prosperity." Reference image(s) here.


54. Consider the following statements about 'the Charter Act of 1813' :

  1. It ended the trade monopoly of the East India Company in India except for trade in tea and trade with China.
  2. It asserted the sovereignty of the British Crown over the Indian territories held by the Company.
  3. The revenues of India were now controlled by the British Parliamen

Which of the statements given above are correct?

  1. 1 and 2 only
  2. 2 and 3 only
  3. 1 and 3 only
  4. 1, 2 and 3

Answer is (a). The East India Company Act 1813, also known as the Charter Act 1813, was an Act of the Parliament of the United Kingdom which renewed the charter issued to the British East India Company, and continued the Company's rule in India. However, the Company's commercial monopoly was ended, except for the tea and opium trade and the trade with China, thus reflecting the growth of British power in India.

  1. The Act expressly asserted the Crown's sovereignty over British India.
  2. It allotted Rs 100,000 to promote education in Indian masses.
  3. This act permitted Christian missionaries to propagate English and preach their religion.

The power of the provincial governments and courts in India over European British subjects was also strengthened by the Act. Financial provision was also made to encourage a revival in Indian literature and for the promotion of science.

Hence, only 1 and 2 are true. The British Parliament took full control only after 1857.


55. With reference to Swadeshi Movement, consider the following statements :

  1. It contributed to the revival of the indigenous artisan crafts and industries.
  2. The National Council of Education was established as a part of Swadeshi Movement.

Which of the statements given above is/are correct?

  1. 1 only
  2. 2 only
  3. Both 1 and 2
  4. Neither 1 nor 2

Answer is (c). Both 1 and 2 are correct. An important aspect of the Swadeshi movement was the emphasis placed on self-reliance or Atmashakti (According to Rabindra Nath Tagore). Several exclusive Indian industrial ventures, Bengal Chemical Swadeshi Stores (Opened by Acharya PC Ray), Lakshmi Cotton Mills, Mohini Mills and National Tannery were started. So statement (1) is true. The National Council of Education (or NCE) was an organisation founded by Indian nationalists in Bengal in 1906 to promote science and technology as part of a swadeshi industrialisation movement. It established the Bengal National College and Bengal Institute which would later merge to form Jadavpur University. Institutions which were functioning under the Council were considered to be hotbeds of swadeshi activities and the government banned nationalistic activities such as the singing of patriotic songs. Statement (2) is also true.


56. Consider the following pairs -

Which of the pairs given above is/are correctly matched?

  1. 1 only
  2. 1 and 2 only
  3. 2 and 3 only
  4. 1, 2 and 3

Answer is (d). All India Anti-Untouchability League was founded by Mahatma Gandhi in 1932. All India Kisan Sabha was founded by Swami Sahajanand Saraswati in 1936. The Self-Respect Movement was founded in 1921 by S.Ramanathan and invited E.V Ramaswamy to head the campaign. E.V Ramaswamy (popularly known as Periyar) is regarded as the ‘Father of Modern Tamil Nadu.’ Hence all statements are true.


57. Which one of the following is not a Harappan site?

  1. Chanhudaro
  2. Kot Diji
  3. Sohgaura
  4. Desalpur

Answer is (c). Sohgaura is not a Harappan site. It is a village on the banks of the Rapti River, about 20km south-east of Gorakhpur, in the Gorakhpur District, Uttar Pradesh where copper plate inscription written in Prakrit in the Brahmi script was discovered. The inscription is sometimes presented as pre-Ashokan, even pre-Mauryan, but the writing of the plate, especially the configuration of akshara would rather suggest a date after Ashoka. The other three – Chanhudaro, Kot Diji and Desalpur are well-known Indus Valley civilisation sites. Reference image(s) here.


58. In which of the following relief sculpture inscriptions is 'Ranyo Ashoka' (King Ashoka) mentioned along with the stone portrait of Ashoka?

  1. Kanganahalli
  2. Sanchi
  3. Shahbazgarhi
  4. Sohgaura

Answer is (a). Near the little known village of Kanganahalli in Karnataka, are the remains of an exceptional Buddhist stupa. Known today after that village, Kanganahalli was rediscovered as a Buddhist site in 1954 and excavated in the 1990s by the Archaeological Survey of India (ASI). Sublime panels illustrate the Buddha’s life from his mother Maya’s dream to his enlightenment. Representations of Yakshis and Nagas and the portrait sculpture of kings, including the first labelled frieze depicting the Mauryan emperor Ashoka, are among its treasures. And then, there is a singular series of animals — winged lions, elephants, camels, deer and bulls — moving in one direction, as if they were circumambulating the sacred chaitya there. Reference image(s) here.


59. Consider the following:

  1. Deification of the Buddha
  2. Treading the path of Bodhisattvas
  3. Image worship and rituals

Which of the above is/are the feature/ features of Mahayana Buddhism?

  1. 1 only
  2. 1 and 2 only
  3. 2 and 3 only
  4. 1, 2 and 3

Answer is (b). Consider some major characteristics of Mahayana Buddhism. 1 and 2 are indeed true, but 3 is not. Although it is customary amongst Buddhists to keep Buddha images and to pay their respects to the Buddha, Buddhists are not idol worshippers. Buddhists revere the image of the Buddha as a gesture to the greatest, wisest, most benevolent, compassionate and holy man who has ever lived in this world. The image is a visual aid that helps one to recall the Buddha in the mind and to remember His great qualities which inspired millions of people from generation to generation throughout the civilized world. So the Buddha is deified, but not worshipped. Bodhisattva is an enlightened being who vows to remain in samsāra (any level) to help all sentient beings achieve enlightenment; characterized by wisdom and compassion.


60. With reference to forced labour (Vishti) in India during the Gupta period, which one of the following statements is correct?

  1. It was considered a source of income for the State, a sort of tax paid by the people.
  2. It was totally absent in the Madhya Pradesh and Kathiawar regions of the Gupta Empire.
  3. The forced labourer was entitled to weekly wages.
  4. The eldest son of the labourer was sent as the forced labourer.

Answer is (a). "Vishti" means forced labour when the labourers are not paid the wages due to them. In Gupta Period, in central and western India, the villagers were subjected to this by the royal army and officials. The position of peasants was undermined from the Gupta period onwards on account of the imposition of forced labour (Vishti) and several new levies and taxes. The peasants had to feed the royal army when it passed their area. Vishti or forced labour for the royal army was also to be provided.

download scanned papers here - Paper 1 Paper II


61. Which one of the following groups of plants was domesticated in the 'New World' and introduced into the 'Old World'?

  1. Tobacco, cocoa and rubber
  2. Tobacco, cotton and rubber
  3. Cotton, coffee and sugarcane
  4. Rubber, coffee and wheat

Answer is (a). New World crops were those that were grown only in the Americas (north and south), before the Europeans reached there, and then took the crops with them. Option (a) is correct as tobacco, cocoa and rubber are new world crops. In option (b), cotton is mentioned which was surely grown in the New World, but also in the Old World. The earliest evidence of cotton use in the Indian subcontinent has been found at the site of Mehrgarh and Rakhigarhi where cotton threads have been found preserved in copper beads; these finds have been dated to the Neolithic (5th millennium BC). So options (b) and (c) are wrong. Wheat was an Old World crop. Common Old World crops (e.g., barley, lentils, oats, peas, rye, wheat), and domesticated animals (e.g., cattle, chickens, goats, horses, pigs, sheep) did not exist in the Americas until they were introduced by post-Columbian contact in the 1490s (as part of the "Columbian Exchange"). So, (d) is wrong. Reference image(s) here.


62. Consider the following statements :

  1. Asiatic lion is naturally found in India only.
  2. Double-humped camel is naturally found in India only.
  3. One-horned rhinoceros is naturally found in India only.

Which of the statements given above is / are correct?

  1. 1 only
  2. 2 only
  3. 1 and 3 only
  4. 1, 2 and 3

Answer is (a). Asiatic lion is now found only in the Gir sanctuary, Gujarat, India. So 1 is correct. So, option (b) is wrong. Double-humped camel – the Bactrian camel (Camelus bactrianus) – is a large, even-toed ungulate native to the steppes of Central Asia. It is found in India also. So, 2 is wrong (“only”). So, options (b) and (d) are wrong. One-horned rhino is found in Nepal also, hence 3 is wrong. So, answer is (a).


63. Consider the following pairs

Which of the pairs given above are correctly matched?

  1. 1 and 2 only
  2. 2 and 3 only
  3. 1 and 3 only
  4. 1, 2 and 3

Answer is (a). Chandrabhaga is one of the major rivers in Amravati district of Maharashtra. As a tributary of the Purna (Bhima), it forms a part of the Tapti-Purna river system. The river is referred to as Chandrabhaga River, especially at Pandharpur, as it resembles the shape of the Moon. The two major rivers draining Tiruchirappalli are the Kaveri and its tributary the Kollidam, but the city is also drained by the Uyyakondan Channel. Hampi is situated on the banks of the Tungabhadra river, and not the Malaprabha river. Bennihalla, Hirehalla and Tuparihalla are the major tributaries to Malaprabha of Dharwad district.


64. In a given year in India, official poverty lines are higher in some States than in others because

  1. poverty rates vary from State to State
  2. price levels vary from State to State
  3. Gross State Product varies from State to State
  4. quality of public distribution varies from State to State

Answer is (b). Option (a) is a wrong explanation. Poverty rates cannot determine poverty lines, but poverty lines will determine poverty rates. Option (b) is correct. The consumption levels will depend on price levels, and accordingly poverty lines are defined differently. Option (c) is wrong as GSDP is not taken into account while deciding poverty lines. Option (d) is not right as PDS quality can impact poverty but that is not used to decide upon the poverty line definition. Reference image(s) here.


65. In the context of which of the following do some scientists suggest the use of cirrus cloud thinning technique and the injection of sulphate aerosol into stratosphere?

  1. Creating the artificial rains in some regions
  2. Reducing the frequency and intensity of tropical cyclones
  3. Reducing the adverse effects of solar wind on the Earth
  4. Reducing the global warming

Answer is (d). Cooling the Earth through the injection of sulphate into the stratosphere is one of the most discussed geo-engineering (GE) schemes. Stratospheric aerosols can sediment into the troposphere, modify the aerosol composition and thus might impact cirrus clouds. It is estimated that enhanced stratospheric aerosol loadings as proposed by several GE approaches will likely lead to a reduced ice crystal nucleation rate and thus optically thinner cirrus clouds. These optically thinner cirrus clouds exert a strong negative cloud forcing in the long-wave which contributes by 60% to the overall net GE forcing. Overall global warming can be reduced.


66. In the context of which one of the following are the terms 'pyrolysis and plasma gasification' mentioned?

  1. Extraction of rare earth elements
  2. Natural gas extraction technologies
  3. Hydrogen fuel-based automobiles
  4. Waste-to-energy technologies

Answer is (d). Controversies surrounding environmental pollution due to poor management of factories using Pyrolysisprocesses have hit headlines recently. Pyrolysis is the thermal decomposition of materials at elevated temperatures in an inert atmosphere. Pyrolysis is used to turn waste plastics (tyres etc.) to oil, through combustion without oxygen. Pyrolysis has been used for turning wood into charcoal since ancient times. Similarly, Plasma gasification is an extreme thermal process using plasma which converts organic matter into a syngas which is primarily made up of hydrogen and carbon monoxide. A plasma torch powered by an electric arc is used to ionize gas and catalyze organic matter into syngas, with slag remaining as a byproduct. Both are “waste-to-energy” technologies. Reference image(s) here.


67. Which of the following are in Agasthyamala Biosphere Reserve

  1. Neyyar, Peppara and Shendurney Wildlife Sanctuaries; and Kalakad Mundanthurai Tiger Reserve
  2. Mudumalai, Sathyamangalam and Wayanad Wildlife Sanctuaries; and Silent Valley National Park
  3. Kaundinya, Gundla Brahme-swaram and Papikonda Wildlife Sanctuaries; and Mukurthi National Park
  4. Kawal and Sri Venkateswara Wildlife Sanctuaries; and Nagarjunasagar-Srisailam Tiger Reserve

Answer is (a). India's Agasthyamala Biosphere Reserve in the Western Ghats was among the 20 new sites added by the UN's top cultural body Unesco to its World Network of Biosphere Reserves, in 2016. It was established in year 2001 and is spread over Kerala and TN. A tough question, that expected you to have remembered the details of each of these Reserves.
A useful page is here - http://civils.pteducation.com/p/protectedareasinindia.html


68. Consider the following statements:

  1. Some species of turtles are herbivores.
  2. Some species of fish are herbivores.
  3. Some species of marine mammals are herbivores.
  4. Some species of snakes are viviparous.

Which of the statements given above are correct?

  1. 1 and 3 only
  2. 2, 3 and 4 only
  3. 2 and 4 only
  4. 1, 2, 3 and 4

Answer is (d). We know for sure that some marine mammals are herbivores (like sea cow – manatee). So 3 is correct, and option (c) is wrong. Boa constrictors and green anacondas are two examples of viviparous snakes, meaning they give birth to live young with no eggs involved at any stage of development. So 4 is correct. So, option (a) is also wrong. Now check 1. Some turtles are indeed herbivores. Hence, option (d) is correct.


69. Consider the following pairs :


Which of the pairs given above are correctly matched?

  1. 1 and 2 only
  2. 2 and 3 only
  3. 1 and 3 only
  4. 1, 2 and 3

Answer is (c). The biggest clue was “Irrawaddy Dolphin”. It cannot be found in Chambal river, but only in eastern part of India or in Irrawaddy river proper (Myanmar). So, 2 is wrong. So, option (c) is right! Rusty-spotted cat is the smallest wild cat. Reference image(s) here.


70. Why is there a great concern about the 'microbeads' that are released into environment?

  1. They are considered harmful to marine ecosystems.
  2. They are considered to cause skin cancer in children.
  3. They are small enough to be absorbed by crop plants in irrigated fields.
  4. They are often found to be used as food adulterants.

Answer is (a). Microbeads are manufactured solid plastic particles of less than one millimeter in their largest dimension. They are most frequently made of polyethylene but can be of other petrochemical plastics such as polypropylene and polystyrene. The past few years have seen a major controversy and concern arising out of these “microplastics” finding their way into marine ecology, and inside the bodies of marine animals.


download scanned papers here - Paper 1 Paper II


71. Building 'Kalyaana Mandapas' was a notable feature in the temple construction in the kingdom of

  1. Chalukya
  2. Chandela
  3. Rashtrakuta
  4. Vijayanagara

Answer is (d). Vijayanagara temples are usually surrounded by a strong enclosure. Medium-sized temples have a garbhagriha, shukanasi (antechamber), a navaranga (antrala) connecting the sanctum and outer mandapa (hall), and a rangamantapa (enclosed pillared hall). In addition to these structures, medium-size temples have a closed circumambulatory (Pradakshinapatha) passage around the sanctum, an open mahamantapa (large hall), a kalyanamandapa (ceremonial hall)and a temple tank to serve the needs of annual celebrations.


72. Consider the following statements :

  1. In the revenue administration of Delhi Sultanate, the in-charge of revenue collection was known as 'Amil'.
  2. The Iqta system of Sultans of Delhi was an ancient indigenous institution.
  3. The office of 'Mir Bakshi' came into existence during the reign of Khalji Sultans of Delhi.

Which of the statements given above is/are correct?

  1. 1 only
  2. 1 and 2 only
  3. 3 only
  4. 1, 2 and 3

Answer is (a). In the Delhi Sultanate, the important officials of a pargana were the amil, the mushrif (also known as amin or munsif the treasurer), the qanungo and two karkuns (clerks). The Central government appointed 'Amil' or 'revenue-collector' in each sub-division called the 'Shiq'. He collected the revenue with the help of hereditary officers of the village like chaudharis, muqaddams, patwaris, etc. So statement (1) is true. The Iqta system was first introduced by Illtutmish in the Delhi sultanate. So statement (2) is not true (it wasn’t an ancient local institution). Office of 'Mir Bakshi' was introduced by Akbar the Mughal King. So statement (3) is also not true.


73. Consider the following statements :

  1. Saint Nimbarka was a contemporary of Akbar.
  2. Saint Kabir was greatly influenced by Shaikh Ahmad Sirhindi.

Which of the statements given above is/are correct?

  1. 1 only
  2. 2 only
  3. Both 1 and 2
  4. Neither 1 nor 2

Answer is (d). Saint Nimbarka was a Hindu philosopher and commentator, known for propagating the Vaishnava doctrine of bhedabheda dvaitadvaita, duality in unity. There is considerable disagreement regarding the dates when Nimbarka lived and taught; according to the Vedic scriptures, he was born in 3096 B.C.E., but modern historical research places him in the thirteenth or fourteenth century. So statement (1) is not true. Some historians believe Kabir's life span from 1398 CE – 1448 CE, while others favour 1440 CE–1518 CE. Shaikh Ahmad al-Sirhindī (1564 CE –1624 CE) was an Islamic scholar, a Hanafi jurist, and a prominent member of the Naqshbandī Sufi order during the Mughal period. Clearly Kabir can't be influenced by the Shaikh Ahmad Sirhind. Statement (2) is not true.


74. With reference to the British colonial rule in India, consider the following statements:

  1. Mahatma Gandhi was instrumental in the abolition of the system of 'indentured labour'.
  2. In Lord Chelmsford's 'War Conference', Mahatma Gandhi did not support the resolution on recruiting Indians for World War.
  3. Consequent upon the breaking of Salt Law by Indian people, the Indian National Congress was declared illegal by the colonial rulers.

Which of the statements given above are correct?

  1. 1 and 2 only
  2. 1 and 3 only
  3. 2 and 3 only
  4. 1, 2 and 3

Answer is (b). The Indian indenture system was an ongoing system a form of debt bondage, by which over a million Indians were transported to various colonies of European powers to provide labour for the plantations, especially sugar. It started from the end of slavery in 1833 and continued until 1920. Mahatma Gandhi was indeed instrumental in the abolition of the system of 'indentured labour'. He saw firsthand the plight of Asian indentured labourers in South Africa and campaigned on this issue during the first decade of the 20th century. The system of indentured labour was officially abolished by British government in 1917.So, 1 is correct. . Lord Chelmsford, Viceroy of India from 1916 to 1921, invited Gandhi to Delhi at a War Conference. In order to gain the trust of the empire, Gandhi agreed to move people to enlist in the army for World War I. However, he wrote to the Viceroy and said that he "personally will not kill or injure anybody, friend or foe". So statement (2) is not correct. After Salt Satyagraha in 1930, in 1932, the British Government declared Indian National Congress, illegal and arrested Gandhiji again. So statement (3) is correct.


75. With reference to Indian National Movement, consider the following pairs :


Which of the pairs given above is/are correctly matched?

  1. 1 only
  2. 1 and 2 only
  3. 3 only
  4. 1, 2 and 3

Answer is (d). All are properly matched. Kshitish Chandra Neogy (1888–1970) as a member of the Constituent Assembly of India, member of the first Cabinet of independent India and the chairman of the first Finance Commission of India. So statement (2) is true. Puran Chand Joshi (14 April 1907 – 9 November 1980), was the first general secretary of the Communist Party of India from 1935–47. So statement (3) is true. "Indian National Liberal Federation" was founded by Surendra Nath Banarjea in 1919, and some of its prominent leaders were Tej Bahadur Sapru, V. S. Srinivasa Sastri and M. R. Jayakar. Tej Bahadur Sapru emerged as the most important leader among the Liberals. So statement (1) is also true.


76. With reference to Mian Tansen, which one of the following statements is not correct?

  1. Tansen was the title given to him by Emperor Akbar.
  2. Tansen composed Dhrupads on Hindu gods and goddesses.
  3. Tansen composed songs on his patrons.
  4. Tansen invented many Ragas.

Answer is (a). Tansen (c. 1500 – 1586), also Ramtanu, was a prominent figure of North Indian (Hindustani) classical music. Actual name of Tansen was Ramtanu. Tansen was the title given to him by the of Gwalior. Tansen was a court musician in the darbar of Raja Ramachandra of Bandavagarh (Rewa). When Akbar heard of his prodigious talent, he sent a ‘firman’ to the king asking for Tansen and made him one of the Navaratnas in his court. He gave him the title of ‘Mian’. All other options are correct. He composed many dhrupads on Ganesha, Shiva, Parvati and Rama. He also composed songs on his patrons. Kalpadruma is a compliation of 300 of his dhrupads that were in Gauhar Bani. Tansen composed in his favourite ragas — Multani, Bhairavi and Todi . He invented the night raga Darbari Kanhra, morning raga Mian Ki Todi, mid-day raga, Mian ki Sarang, seasonal raga Mian ki Malhar. His descendants and disciples are called Seniyas


77. Who among the following Mughal Emperors shifted emphasis from illustrated manuscripts to album and individual portrait?

  1. Humayun
  2. Akbar
  3. Jahangir
  4. Shah Jahan

Answer is (c). A tough question! During the reign of Akbar, the imperial court also emerged as a centre of cultural excellence. Akbar inherited and expanded his father's library and atelier of court painters. Between 1560 and 1566 the Tutinama ("Tales of a Parrot"), was illustrated. As Mughal-derived painting spread to Hindu courts the texts illustrated included the Hindu epics including the Ramayana and the Mahabharata. It changed in the reign of Jahangir, who had an artistic inclination. Brushwork became finer and the colors lighter. Jahangir was deeply influenced by European painting. During his reign he came into direct contact with the English Crown and was sent gifts of oil paintings, which included portraits of the King and Queen. He encouraged his royal atelier to take up the single point perspective favoured by European artists, unlike the flattened multi-layered style used in traditional miniatures. He particularly encouraged paintings depicting events of his own life, individual portraits, and studies of birds, flowers and animals.


78. Which one of the following National Parks lies completely in the temperate alpine zone?

  1. Manas National Park
  2. Namdapha National Park
  3. Neora Valley National Park
  4. Valley of Flowers National Park

Answer is (b). In India the alpine climate zone is found in the Himalayan mountains, where the temperature falls by 0.6 °C for every 100 m rise in altitude and this gives rise to a variety of climates from nearly tropical in the foothills to tundra type above the snow line. The states of Jammu and Kashmir, Himachal Pradesh, Uttarakhand, Arunachal Pradesh, Northern districts of West Bengal and Sikkim experience this kind of weather. Manas National Park is in Assam. Namdapha National Park is in Arunachal Pradesh. Neora Valley National Park is in West Bengal in Kalimpong District and Valley of Flowers National Park is in Uttarakhand. The Valley of Flowers, covering about 87.5 sq.km, is very heterogeneous geographically, varying from the temperate zone (2400-3000m) to sub-alpine forests (3000-3300m), to the alpine zone (3300-3500m). But it lies completely in the temperate alpine zone.


79. Atal Innovation Mission is set up under the

  1. Department of Science and Technology
  2. Ministry of Employment
  3. NITI Aayog
  4. Ministry of Skill Development and Entrepreneurship

Answer is (c). The Atal Innovation Mission (AIM) is a flagship initiative set up by the NITI Aayog to promote innovation and entrepreneurship across India. AlM's objectives are to create and promote an ecosystem of innovation and entrepreneurship across the country at school, university, research institutions, MSME and industry levels.


80. On 21st June, the Sun

  1. does not set below the horizon at the Arctic Circle
  2. does not set below the horizon at Antarctic Circle
  3. shines vertically overhead at noon on the Equator
  4. shines vertically overhead at the Tropic of Capricorn

Answer is (a). On the 21st June or June solstice, the midnight sun is visible (weather permitting) throughout the night, in all areas from just south of the Arctic Circle to the North Pole. On the other side of the planet, south of the Antarctic Circle there's Polar Night, meaning no Sunlight at all, on the June solstice. Reference image(s) here.




81. Consider the following statements:

  1. Agricultural soils release nitrogen oxides into environment.
  2. Cattle release ammonia into environment.
  3. Poultry industry releases reactive nitrogen compounds into environment.

Which of the statements given above is/are correct?

  1. 1 and 3 only
  2. 2 and 3 only
  3. 2 only
  4. 1, 2 and 3

Answer is (d). Statement 2 is definitely correct. Cattle account for 80% of the ammonia production, though their annual growth rate is 1%, due to a stable population. India is globally the biggest source of ammonia emission, nearly double that of NOx emissions. So, option (a) is wrong. Just check 3 now. The poultry industry releases reactive nitrogen compounds into the environment. So, 3 is correct. Now check 1. Agricultural soils contributed to over 70% of N2O emissions from India in 2010, followed by waste water (12%) and residential and commercial activities (6%). Since 2002, N2O has replaced methane as the second largest Greenhouse Gas (GHG) from Indian agriculture. So all statements are right. So, option (d) is correct.


82.What is common to the places known as Aliyar, Isapur and Kangsabati?

  1. Recently discovered uranium deposits
  2. Tropical rain forests
  3. Underground cave systems
  4. Water reservoirs

Answer is (d). These are all places with water reservoirs, in South India. They were in news due to constantly dipping water volumes.


83.In the context of proposals to the use of hydrogen-enriched CNG (H-CNG) as fuel for buses in public transport, consider the following statements:

  1. The main advantage of the use of H-CNG is the elimination of carbon monoxide emissions.
  2. H-CNG as fuel reduces carbon dioxide and hydrocarbon emissions.
  3. Hydrogen up to one-fifth by volume can be blended with CNG as fuel for buses.
  4. H-CNG makes the fuel less expensive than CNG.

Which of the statements given above is / are correct?

  1. 1 only
  2. 2 and 3 only
  3. 4 only
  4. 1, 2, 3 and 4

Answer is (b). Statement 1 is wrong, as HCNG does not “eliminate” carbon monoxide emissions. So, options (a) and (d) are wrong. Look at Statement 2. It is correct. So, we only have option (b) left. That is our answer.


84.Why are dewdrops not formed on a cloudy night?

  1. Clouds absorb the radiation released from the Earth's surface.
  2. Clouds reflect back the Earth's radiation.
  3. The Earth's surface would have low temperature on cloudy nights.
  4. Clouds deflect the blowing wind to ground level.

Answer is (b). Dew drops are formed due to condensation of water vapour. The air around us contains water vapour that is known as moisture or humidity. Hot air contains more moisture as compared to cold air. During the night when hot air comes into contact with some cold surface, water vapour present in it condenses on the cold surface in the form of droplets. These tiny drops of water are called dew drops. Dew formation is more when the sky is clear and less when it is cloudy. When the sky is clear and the trees and plants are cooler at nights, there is more evaporation of water and hence more dew formation. But when it is cloudy, trees and plants do not get cool in the night (clouds reflecting back the radition) and hence there is less dew formation. Reference image(s) here.


85.Consider the following statements:

  1. The 44th Amendment to the Constitution of India introduced an Article placing the election of the Prime Minister beyond judicial review.
  2. The Supreme Court of India struck down the 99th Amendment to the Constitution of India as being violative of the independence of judiciary.

Which of the statements given above is/are correct?

  1. 1 only
  2. 2 only
  3. Both 1 and 2
  4. Neither 1 nor 2

Answer is (b). The Constitution (Forty-fourth Amendment) Act, 1978 actually undid a lot of changes that the 42nd amendment 1976 had brought in, during the emergency period. Hence, statement 1 is wrong. Statement 2 is correct. It refers to the creation of the National Judicial Appointments Commission (NJAC) which was struck down by the SC in 2015.


86.Consider the following statements:

  1. The- motion to impeach a Judge of the Supreme Court of India cannot be rejected by the Speaker of the Lok Sabha as per the Judges (Inquiry) Act, 1968.
  2. The Constitution of India defines and gives details of what Constitutes 'incapacity and proved misbehaviour' of the Judges of the Supreme Court of India.
  3. The details of the process of impeachment of the Judges of the Supreme Court of India are given in the Judges (Inquiry) Act, 1968.
  4. If the motion for the impeachment of a Judge is taken up for voting, the law requires the motion to be backed by each House of the Parliament and supported by a majority of total membership of that House and by not less than two-thirds of total members of that House present and voting.

Which of the statements given above is/are correct?

  1. 1 and 2
  2. 3 only
  3. 3 and 4 only
  4. 1, 3 and 4

Answer is (c). Statement 1 is wrong because the Speaker may admit the motion, or refuse to admit it. So options (a) and (d) are gone. Check statement 4 now. It is indeed correct. The Constitution has measures to ensure the independence of the judiciary from executive action. This helps judges give judicial decisions in a free and fair manner without any inducements. The Constitution also provides checks against misbehaviour by judges. It states that a judge may be removed only through a motion in Parliament with a two thirds support in each House. The process is laid down in the Judges (Inquiry) Act, 1968. The motion requires two-third majority of members present and voting of that house and an absolute majority of its total membership of that house.


87.The Ninth Schedule was introduced in the Constitution of India during the prime ministership of

  1. Jawaharlal Nehru
  2. Lal Bahadur Shastri
  3. Indira Gandhi
  4. Morarji Desai

Answer is (a). The Ninth Schedule was the product of the first-ever amendment to the Indian Constitution. It was introduced by the Nehru Government, on 10 May 1951 to address judicial decisions and pronouncements especially about the chapter on fundamental rights. As a result of these judicial pronouncements, the Government was apprehensive that the whole agrarian reform programmes would be endangered. Article 31-B was inserted by the First Constitutional (Amendment) Act 1951 which states that without prejudiced to the generality of the provisions contained in Article 31-A, none of the Acts and Regulations specified in the Ninth Schedule nor any of the provisions thereof shall be deemed to be void, etc. Thus Article 31-B of the Constitution of India ensured that any law in the Ninth Schedule could not be challenged in courts and Government can rationalize its programme of social engineering by reforming land and agrarian laws. In other words laws under Ninth Schedule are beyond the purview of judicial review even though they violate fundamental rights enshrined under part III of the Constitution. On the one hand considerable power was given to legislature under Article 31-B and on the other hand the power of judiciary was curtailed.


88.Consider the following statements:

  1. Coal sector was nationalized by the Government of India under Indira Gandhi.
  2. Now, coal blocks are allocated on lottery basis.
  3. Till recently, India imported coal to meet the shortages of domestic supply, but now India is self-sufficient in coal production.

Which of the statements given above is/are correct?

  1. 1 only
  2. 2 and 3 only
  3. 3 only
  4. 1, 2 and 3

Answer is (a). Statement 1 is indeed correct, and Indira Gandhi’s government nationalized coal. So, options (b) and (c) are wrong. Now check 2. It is wrong as allocation are made through competitive bidding process. Hence, answer is (a).


89.Consider the following statements:

  1. The Parliament (Prevention of Disqualification) Act, 1959 exempts several posts from disqualification on the grounds of 'Office of Profit'.
  2. The above-mentioned Act was amended five times.
  3. The term 'Office of Profit' is well-defined in the Constitution of India.

Which of the statements given above is/are correct?

  1. 1 and 2 only
  2. 3 only
  3. 2 and 3 only
  4. 1, 2 and 3

Answer is (a). Statement 3 is interesting – it is wrong! So all options are eliminated but (a). The term office of profit I not explicitly defined in the Indian constitution. That is why the Parliament (Prevention of Disqualification) Act, 1959 was enacted (to bring clarity). So, what is the concept of ‘office of profit’? MPs and MLAs, as members of the legislature, hold the government accountable for its work. The essence of disqualification under the office of profit law is if legislators holds an ‘office of profit’ under the government, they might be susceptible to government influence, and may not discharge their constitutional mandate fairly. What constitutes an ‘office of profit’? The law does not clearly define what constitutes an office of profit but the definition has evolved over the years with interpretations made in various court judgments. An office of profit has been interpreted to be a position that brings to the office-holder some financial gain, or advantage, or benefit. The amount of such profit is immaterial. In 1964, the Supreme Court ruled that the test for determining whether a person holds an office of profit is the test of appointment. Several factors are considered in this determination including factors such as: (i) whether the government is the appointing authority, (ii) whether the government has the power to terminate the appointment, (iii) whether the government determines the remuneration, (iv) what is the source of remuneration, and (v) the power that comes with the position. What is the Constitutional position? Under the provisions of Article 102 (1) and Article 191 (1) of the Constitution, an MP or an MLA (or an MLC) is barred from holding any office of profit under the central or state government. The articles clarify that “a person shall not be deemed to hold an office of profit under the government of India or the government of any state by reason only that he is a minister”. The Constitution specifies that the number of ministers including the Chief Minister has to be within 15% of the total number of members of the assembly (10% in the case of Delhi, which is a union territory with legislature).

download scanned papers here - Paper 1 Paper II


90.Under which Schedule of the Constitution of India can the transfer of tribal land to private parties for mining be declared null and void?

  1. Third Schedule
  2. Fifth Schedule
  3. Ninth Schedule
  4. Twelfth Schedule

Answer is (b). In the Article 244(1) of the Constitution, expression Scheduled Areas means such areas as the President may by order declare to be Scheduled Areas. The Scheduled areas can be established under Article 244 and 5th Schedule of the Constitution, and are for safeguarding tribal interests.


91.Recently, there was a growing awareness in our country about the importance of Himalayan nettle (Girardinia diversifolia) because it is found to be a sustainable source of

  1. anti-malarial drug
  2. blodiesel
  3. pulp for paper industry
  4. textile fibre

Answer is (d). Girardinia diversifolia, commonly known as the Himalayan Nettle is a naturally available wild shrub found abundantly in the foothills of Himalayas. According to the Centre for Sustainable Fashion, Himalayan Nettle is the longest fiber and is considered finer, stronger, and more elastic than linen. This new trend to use the Nettle fiber for textile purpose has been driven by concern over the environmental damage caused by the processing of synthetic fibers and fabrics.


92.For the measurement/estimation of which of the following are satellite images/remote sensing data used?

  1. Chlorophyll content in the vegetation of a specific location.
  2. Greenhouse gas emissions from rice paddies of a specific location.
  3. Land surface temperatures of a specific location.

Select the correct answer using the code given below.

  1. 1 only
  2. 2 and 3 only
  3. 3 only
  4. 1, 2 and 3

Answer is (d). All these applications are possible using the present generation satellites.


93.Consider the following States:

  1. Chhattisgarh
  2. Madhya Pradess
  3. Maharashtra
  4. Odisha

With reference to the States mentioned above, in terms of percentage of forest cover to the total area of State, which one of the following is the correct ascending order?

  1. 2-3-1-4
  2. 2-3-4-1
  3. 3-2-4-1
  4. 3-2-1-4

Answer is (c).1. Chhattisgarh - 41.09%, 2. Madhya Pradesh - 25.11%, 3. Maharashtra - 16.47%, 4. Odisha - 32.98%. Thus, the order will be 3-2-4-1.


94.Which of the following statements are correct about the deposits of 'methane hydrate'?

  1. Global warming might trigger the release of methane gas from these deposits.
  2. Large deposits of 'methane hydrate' are found in Arctic Tundra and under the seafloor.
  3. Methane in atmosphere oxidizes to carbon dioxide after a decade or two.

Select the correct answer using the code given below.

  1. 1 and 2 only
  2. 2 and 3 only
  3. 1 and 3 only
  4. 1, 2 and 3

Answer is (d). Methane is relatively short-lived in the atmosphere; a molecule of methane is oxidized to water and carbon dioxide within a decade or so. Statement (3) is correct. So (a) can't be the answer. Methane hydrate is a cage-like lattice of ice inside of which are trapped molecules of methane, the chief constituent of natural gas. If methane hydrate is either warmed or depressurized, it will revert back to water and natural gas. Hydrate deposits may be several hundred meters thick and generally occur in two types of settings: under Arctic permafrost, and beneath the ocean floor. Statement (2) is also correct. Climate warming, however, could cause the hydrates to destabilize. The methane, a potent greenhouse gas, would escape unused into the atmosphere and could even accelerate climate change. Statement (3) is also correct.


95.Consider the following:

  1. Carbon monoxide
  2. Methane
  3. Ozone
  4. Sulphur dioxide

Which of the above are released into atmosphere due to the burning of crop/biomass residue?

  1. 1 and 2 only
  2. 2, 3 and 4 only
  3. 1 and 4 only
  4. 1, 2, 3 and 4

Answer is (d). Burning is an easiest and economical option for management of crop/biomass residues. Due to lack of awareness or not availability of suitable technologies it is generally practices everywhere. Burning of crop residues not only degrade the atmospheric quality but also affect the climate and ultimate the human health. Crop residue and biomass burning (forest fires) are considered as a major source of carbon dioxide (CO2), carbon monoxide (CO), methane (CH4), volatile organic compounds (VOC), nitrogen oxides, non-methane hydrocarbons (NMHCs), ozone (O3), and aerosols etc. Reference image(s) here.


96.Consider the following pairs:

Sea - Bordering country

  1. Adriatic Sea - Albania
  2. Black Sea - Croatia
  3. Caspian Sea - Kazakhstan
  4. Mediterranean Sea - Morocco
  5. Red Sea - Syria

Which of the pair given above are correctly matched?

  1. 1, 2 and 4 only
  2. 1, 3 and 4 only
  3. 2 and 5 only
  4. 1, 2, 3, 4 and 5

Answer is (b). Syria is nowhere touching the Red Sea. So 5 is wrong, and so options (c) and (d) are wrong. Caspian Sea - Kazakhstan is correct. So answer could be (b) only. Also, Croatia is bordering Mediterranean sea and not the Black sea. Adriatic Sea - Albania is correct. So option (b) is correct. Reference image(s) here.


97.Among the following, which one is the largest exporter of rice in the world in the last five years?

  1. China
  2. India
  3. Myanmar
  4. Vietnam

Answer is (b). India has become the world's largest rice exporter in the last five years. But the export of rice has declined in this current year because the quality of rice in India is getting worse due to excessive use of pesticides and therefore there has been a decline in export. But now perhaps these conditions may improve.


98.Consider the following pairs:

Glacier -- River

  1. Bandarpunch -- Yamuna
  2. Bara Shigri -- Chenab
  3. Milam -- Mandakini
  4. Siachen -- Nubra
  5. Zemu -- Manas

Which of the pairs given above are correctly matched?

  1. 1, 2 and 4
  2. 1, 3 and 4
  3. 2 and 5
  4. 3 and 5

Answer is (c). Yamuna river originates from Yamunotri Glacier. Bandarpuch is a Massif not a Glacier. So, 1 is wrong, and answer can be (c) or (d). Chenab river originates from Bara Shigri Glacier. So answer is (c). Mandakini river originates from Chorabari Glacier and Nubra river originates from Rimo Glacier.


99.In India, the use of carbofuran, methyl parathion, phorate and triazophos is viewed with apprehension. These chemicals are used as

Glacier -- River

  1. pesticides in agriculture
  2. preservatives in processed foods
  3. fruit-ripening agents
  4. moisturising agents in cosmetics

Answer is (a). To promote organic farming in Kerala, the state agriculture department had ordered a ban on the use of two categories of pesticides on May 7, 2015. The order was to be implemented within 10 days. During this period the Kerala Agriculture University was asked to provide alternatives to the banned pesticides, which include carbofuran, phorate, methyl parathion, monocrotophos, methyl demethon, prophenophos and triazophos.


100.Consider the following statements :

  1. Under Ramsar Convention, it is mandatory on the part of the Government of India to protect and conserve all the wetlands in the territory of India.
  2. The Wetlands (Conservation and Management) Rules, 2010 were framed by the Government of India based on the recommendations of Ramsar Convention.
  3. The Wetlands (Conservation and Management) Rules, 2010 also encompass the drainage area or catchment regions of the wetlands as determined by the authority.

Which of the statements given above is / are correct?

  1. 1 and 2 only
  2. 2 and 3 only
  3. 3 only
  4. 1, 2 and 3

Answer is (c). Wetlands (Conservation and Management) Rules, 2010 have been drafted by the Ministry of Environment and Forests to ensure better conservation and management and to prevent degradation of existing wetlands in India. They are not based on Ramsar convention. So, statement 2 is wrong. So (a), (c) and (d) are not correct.







Questions and Detailed Solutions are being continuously updated ... refresh and check. Comment and let us know your experience, answers and solutions too!